Clockwise and counterclockwise triangles

Geometry Level 5

In the diagram below, we have 3 equilateral triangles and 3 identical circles with the following properties:

  • The 3 circles centered at A , B , C A, B, C are tangent to one another, and A B C \triangle ABC has area S A B C . S_{ABC}.
  • G J , J I , I G GJ, JI, IG are tangent to circles centered at A , B , C , A, B, C, respectively, and G J I \triangle GJI has area S G J I . S_{GJI}.
  • K L , L H , H K KL, LH, HK are tangent to circles centered at A , B , C , A, B, C, respectively, and K L H \triangle KLH has area S K L H . S_{KLH}.

What is S G J I + S K L H S A B C = P Q , \frac{S_{GJI}+S_{KLH}}{S_{ABC}}=\frac{P}{Q}, where P P and Q Q are coprime positive integers?

Give your answer as P + Q . P+Q.

Note: As a related problem, see 2018 AMC 12B Problem #25 .


The answer is 9.

This section requires Javascript.
You are seeing this because something didn't load right. We suggest you, (a) try refreshing the page, (b) enabling javascript if it is disabled on your browser and, finally, (c) loading the non-javascript version of this page . We're sorry about the hassle.

2 solutions

This is a partial solution

I decided to solve for the area of Δ K H L \Delta KHL only. The area of Δ A B C \Delta ABC is obvious, for the area of Δ G H I \Delta GHI I have a link to a youtube video that explains the solution and is also similar to this approach. Also if you have solved the last question of the AMC 12B, you will probably get this.

Click Here

Let A K B L = X AK \cap BL = X

m A K L = 9 0 m\angle AKL = 90^{\circ} and m B L H = 9 0 m\angle BLH = 90^{\circ} since K L KL and H L HL are tangents.

Also m K L H = 6 0 m\angle KLH = 60^{\circ} thus m K L X = 3 0 m\angle KLX = 30^{\circ} giving m L K X = 6 0 m\angle LKX = 60^{\circ} and m A K B = 12 0 m\angle AKB = 120^{\circ} .

Let X K = a XK = a , thus K L = a 3 KL = a\sqrt{3} and X L = 2 a . XL=2a. Also let A K = B L = b AK=BL=b then A B = 2 b AB=2b , A X = b a AX = b-a and X B = 2 a + b XB=2a+b .

By Law of Cosines on A X B \angle AXB , we get ( 2 b ) 2 = ( b a ) 2 + ( 2 a + b ) 2 2 ( b a ) ( 2 a + b ) cos 120 {(2b)}^2 = {(b-a)}^2 + {(2a+b)}^2 -2(b-a)(2a+b)\cos120 .

Solving for a a in terms of b b , we get a = b 21 3 b 6 . a= \dfrac{b\sqrt{21} - 3b}{6}. Since K L = L H = K H = a 3 = b 63 3 3 b 6 KL=LH=KH=a\sqrt{3}= \dfrac{b\sqrt{63} - 3\sqrt{3}b}{6}

we get the area of Δ K L H \Delta KLH as 1 2 ( 5 3 3 7 ) b 2 \dfrac{1}{2}({5\sqrt{3} - 3\sqrt{7}})b^2 .

Also we get the area of Δ A B C \Delta ABC and Δ G H I \Delta GHI as 4 b 2 3 4b^2\sqrt{3} and 1 2 ( 5 3 + 3 7 ) b 2 \dfrac{1}{2}({5\sqrt{3} + 3\sqrt{7}})b^2 .

Performing the required division above gives 5 4 \dfrac{5}{4} .

A + B = 9 . \therefore A+B = \boxed{9}.

David Vreken
Mar 16, 2018

To find the area S G J I S_{GJI} of an equilateral triangle G J I \triangle GJI , we will let M M be the intersection of A B AB and I J IJ and examine A J M \triangle AJM and B I M \triangle BIM .

Let r r be the radius of any of the three congruent circles. Then A J = B I = r AJ = BI = r and A B = 2 r AB = 2r . Since I J IJ is tangent to circle B B , B I J = 90 ° \angle BIJ = 90° . Since J G JG is tangent to circle A A , A J G = 90 ° \angle AJG = 90° , and since I J G \triangle IJG is an equilateral triangle, I J G = 60 ° \angle IJG = 60° , which means A J I \angle AJI = = A J G I J G \angle AJG - \angle IJG = = 90 ° 60 ° 90° - 60° = = 30 ° 30° .

By law of sines on A J M \triangle AJM , sin A M J A J = sin A J M A M \frac{\sin \angle AMJ}{AJ} = \frac{\sin \angle AJM}{AM} or sin A M J r = sin 30 ° A M \frac{\sin \angle AMJ}{r} = \frac{\sin 30°}{AM} , and by law of sines on B I M \triangle BIM , sin B M I B I = sin B I M M B \frac{\sin \angle BMI}{BI} = \frac{\sin \angle BIM}{MB} or sin B M I r = sin 90 ° M B \frac{\sin \angle BMI}{r} = \frac{\sin 90°}{MB} . Since A M J = B M I \angle AMJ = \angle BMI as vertical angles, we can combine the two ratios and arrive at sin 30 ° A M = sin 90 ° M B \frac{\sin 30°}{AM} = \frac{\sin 90°}{MB} , and simplifying this gives M B = 2 A M MB = 2AM . Since A M + M B = A B = 2 r AM + MB = AB = 2r , A M + 2 A M = 2 r AM + 2AM = 2r , so A M = 2 3 r AM = \frac{2}{3}r , and M B = 2 A M = 2 2 3 r = 4 3 r MB = 2AM = 2\frac{2}{3}r = \frac{4}{3}r .

Using sin A M J r = sin 30 ° A M \frac{\sin \angle AMJ}{r} = \frac{\sin 30°}{AM} once again along with A M = 2 r AM = 2r , we find that A M J = sin 1 3 4 \angle AMJ = \sin^{-1}\frac{3}{4} . Since the sum of the angles of A J M \triangle AJM is 180 ° 180° , M A J \angle MAJ = = 180 ° A J M A M J 180° - \angle AJM - \angle AMJ = = 180 ° 30 ° sin 1 3 4 180° - 30° - \sin^{-1}\frac{3}{4} = = 150 ° sin 1 3 4 150° - \sin^{-1}\frac{3}{4} . Therefore, sin M A J \sin \angle MAJ = = sin ( 150 ° sin 1 3 4 ) \sin (150° - \sin^{-1}\frac{3}{4}) = = sin 150 ° cos ( sin 1 3 4 ) cos 150 ° sin ( sin 1 3 4 ) \sin 150° \cos(\sin^{-1}\frac{3}{4}) - \cos 150° \sin(\sin^{-1}\frac{3}{4}) = = 7 + 3 3 8 \frac{\sqrt{7} + 3\sqrt{3}}{8} .

My law of sines on A J M \triangle AJM once again, sin A J M A M = sin M A J M J \frac{\sin \angle AJM}{AM} = \frac{\sin \angle MAJ}{MJ} or sin 30 ° 2 3 r = 7 + 3 3 8 M J \frac{\sin 30°}{\frac{2}{3}r} = \frac{\frac{\sqrt{7} + 3\sqrt{3}}{8}}{MJ} , which means M J = ( 7 + 3 3 6 ) r MJ = (\frac{\sqrt{7} + 3\sqrt{3}}{6})r .

My Pythagorean's Theorem, M I = M B 2 B I 2 = ( 4 3 r ) 2 r 2 = 7 3 r MI = \sqrt{MB^2 - BI^2} = \sqrt{(\frac{4}{3}r)^2 - r^2} = \frac{\sqrt{7}}{3}r .

Since I J = M I + M J IJ = MI + MJ , I J IJ = = 7 3 r + ( 7 + 3 3 6 ) r \frac{\sqrt{7}}{3}r + (\frac{\sqrt{7} + 3\sqrt{3}}{6})r = = 7 + 3 2 r \frac{\sqrt{7} + \sqrt{3}}{2}r .

The area S G J I S_{GJI} of an equilateral triangle G J I \triangle GJI formed by side lengths I J IJ would then be A A = = 3 4 ( 7 + 3 2 r ) 2 \frac{\sqrt{3}}{4}(\frac{\sqrt{7} + \sqrt{3}}{2}r)^2 = = 5 3 + 3 7 8 r 2 \frac{5\sqrt{3} + 3\sqrt{7}}{8}r^2 .


To find the area S K L H S_{KLH} of an equilateral triangle K L H \triangle KLH , we will examine A K F \triangle AKF .

If r r is the radius of any of the three congruent circles, then A K = r AK = r . Also, since A A is the vertex of equilateral triangle A B C \triangle ABC with sides 2 r 2r , and F F is the center of that equilateral triangle, A F = 2 3 3 r AF = \frac{2\sqrt{3}}{3}r . Since K L KL is tangent to circle A A , A K L = 90 ° \angle AKL = 90° , and since K L H \triangle KLH is an equilateral triangle, L K H = 60 ° \angle LKH = 60° , and since F F is the center of equilateral triangle K L H \triangle KLH , L K F = 30 ° \angle LKF = 30° , which means A K F \angle AKF = = A K L + L K F \angle AKL + \angle LKF = = 90 ° + 30 ° 90° + 30° = = 120 ° 120° .

By the law of cosines on A K F \triangle AKF , we have A F 2 = A K 2 + F K 2 2 A K F K cos A K F AF^2 = AK^2 + FK^2 - 2 \cdot AK \cdot FK \cdot \cos \angle AKF or ( 2 3 3 r ) 2 = r 2 + F K 2 2 r F K cos 120 ° (\frac{2\sqrt{3}}{3}r)^2 = r^2 + FK^2 - 2 \cdot r \cdot FK \cdot \cos 120° , which solves to F K = 21 3 6 r FK = \frac{\sqrt{21} - 3}{6}r .

Since K K is the vertex of equilateral triangle K L H \triangle KLH , and F F is the center of that equilateral triangle, K L = 3 F K KL = \sqrt{3}FK . Therefore, since F K = 21 3 6 r FK = \frac{\sqrt{21} - 3}{6}r , K L = 7 3 2 r KL = \frac{\sqrt{7} - \sqrt{3}}{2}r .

The area S K L H S_{KLH} of an equilateral triangle K L H \triangle KLH formed by side lengths K L KL would then be A A = = 3 4 ( 7 3 2 r ) 2 \frac{\sqrt{3}}{4}(\frac{\sqrt{7} - \sqrt{3}}{2}r)^2 = = 5 3 3 7 8 r 2 \frac{5\sqrt{3} - 3\sqrt{7}}{8}r^2 .


Finally, the area S A B C S_{ABC} of an equilateral triangle A B C \triangle ABC formed by side lengths 2 r 2r would be A A = = 3 4 ( 2 r ) 2 \frac{\sqrt{3}}{4}(2r)^2 = = 3 r 2 \sqrt{3}r^2 .


Therefore, S G J I + S K L H S A B C \frac{S_{GJI} + S_{KLH}}{S_{ABC}} = = 5 3 + 3 7 8 r 2 + 5 3 3 7 8 r 2 3 r 2 \frac{\frac{5\sqrt{3} + 3\sqrt{7}}{8}r^2 + \frac{5\sqrt{3} - 3\sqrt{7}}{8}r^2}{\sqrt{3}r^2} = = 5 4 \frac{5}{4} , and 5 + 4 = 9 5 + 4 = \boxed{9} .

Great analysis

Valentin Duringer - 1 year ago

0 pending reports

×

Problem Loading...

Note Loading...

Set Loading...